2 svar
61 visningar
pepsi1968 är nöjd med hjälpen
pepsi1968 481
Postad: 3 mar 2020 21:52

El

 

Hej! Jag undrar lite vad som händer här. I.o.m att det inte finns något batteri i uppgiften så antar jag att spänningen osv blir såhär:

Blir lite knas där i mitten, men stämmer detta?

Ture 9884 – Livehjälpare
Postad: 3 mar 2020 22:10

Den lodräta resistorn i mitten kommer att ha samma spänning på bägge sidor om sig, det går därför ingen ström genom det. (pga symmetri) 

Därför kan det tas bort utan att påverka kretsen i övrigt. Sen är det enkelt att förenkla nätet. 2R parallellt med 2R, 

SaintVenant 3844
Postad: 3 mar 2020 22:25 Redigerad: 3 mar 2020 22:30

Felpost.

Svara Avbryt
Close